15
$\begingroup$

Suppose $R$ is a regular local ring. Let $m$ be the maximal ideal. Then, if the dimension of $R$ is $n$, there is a regular sequence of size $n$, say $x_1,x_2,...,x_n$ s.t. $m=(x_1,x_2,...,x_n)R$. Further, the ideals $(x_{i_1},...,x_{i_j})$ with $i_1,...,i_j\in {1,...,n}$, are prime.

Can we make similar statements about any other kind of prime ideals in a regular local ring $R$? Specifically, do any other prime ideals satisfy the condition: if the ideal is minimally generated by a certain set of generators, then every subset of the generators defines a prime ideal? One example in light of the first paragraph, are the prime ideals generated by a subset of the regular sequence that generates the maximal ideal.

Also, when does a regular sequence define a prime ideal in a regular local ring $R$, and when does a maximal regular sequence define a maximal ideal?

$\endgroup$
4
  • $\begingroup$ Not really worth much but certainly if $R/I$ is regular, then $I$ satisfies the condition you pointed out (because then the generators of $I$ can be made to be part of a regular sequence defining the maximal ideal). $\endgroup$ Feb 7, 2011 at 17:59
  • $\begingroup$ @Karl: Thanks for your response. Does this really give us a new example? Since, $R$ is regular, $R/I$ is regular if and only if $I$ is generated by a regular system of parameters. This takes us back to the family I mention. Am I missing something here? $\endgroup$ Feb 7, 2011 at 20:13
  • $\begingroup$ No no, not a new example at all. Just a different way of phrasing your example. $\endgroup$ Feb 8, 2011 at 3:58
  • $\begingroup$ @Karl: Yes, definitely a more compact (better) way of putting it. $\endgroup$ Feb 8, 2011 at 4:06

2 Answers 2

7
$\begingroup$

As Sandor pointed out, a necessary condition is that the prime ideal $P$ is a complete intersection. Here is a proof that it is also sufficient. It will suffice to prove the following:

Claim: Let $(R,m)$ be a Noetherian local ring and $x\in m$ a regular element on $R$. If $R/(x)$ is a domain, then so is $R$.

Proof: Suppose $ab=0$ in $R$. Then modulo $x$, one of them say $a$, must be $0$. So $a=xa_1$, thus $x(a_1b)=0$. As $x$ is regular, $a_1b=0$, and continuing in this fashion one of $a,b$ must be divisible by arbitrary high power of $x$, so it must be equal to $0$.

As for an example which is not a part of a regular s.o.p, take something like $P=(x^2+y^2+z^2, u^2+v^2+w^2)$ in $\mathbb C[[x,y,z,u,v,w]]$.

$\endgroup$
2
  • $\begingroup$ @Hailong: Thanks for the answer. Just to make it completely explicit. If $P$ is a complete intersection in a regular local ring (hence Cohen Macaulay), there is a regular sequence that generates it. We can reduce to case where $P$ is $2$ generator using induction. Then, using your claim and the fact that regular sequences are permutable in a Noetherian local ring, we get the result. Does this sound OK? $\endgroup$ Feb 8, 2011 at 4:14
  • $\begingroup$ @Koose: I am not sure how to reduce to 2-generator case. What I had in mind is pretty simple: if $P=(x_1,\cdots,x_n)$, then my claim shows that $(x_2,\cdots, x_{n})$ is prime. Also, since one can permute reg sequence, all subsets of min. generators of $P$ generate prime ideals. $\endgroup$ Feb 8, 2011 at 19:55
10
$\begingroup$

EDIT: Rephrased the answer in light of Koose's comment.

Claim. Any ideal satisfying the required property has to be a complete intersection.

Proof. Let $\mathfrak p\subset R$ be a prime ideal. Assume that the minimal number of generators for $\mathfrak p$ is $r$ and let $a_1,\dots,a_r\in\mathfrak p$ be a set of generators. Let $I_t=(a_1,\dots,a_t)$ and one has the sequence of ideals:

$$ 0\subsetneq I_1 \subsetneq \dots \subsetneq I_t\subsetneq I_{t+1}\subsetneq \dots\subsetneq I_r=\mathfrak p $$

The containments cannot be equalities, because that would make the corresponding $a_i$ unneeded to generate $\mathfrak p$. If all the $I_t$'s are prime, then $\mathfrak p$ has height at least $r$, but it cannot be more than that, so the claim is proven. $\square$

Example. Take an irreducible projective variety (say the twisted cubic curve) that is not a complete intersection (in projective space). Then the ideal of the affine cone over this in the affine cone over projective space will be a prime, but if you take away one of the generators, that will not be.

$\endgroup$
3
  • $\begingroup$ @Sandor: Thanks for the answer. I am not familiar with all the terminology here, but this only seems to show that the particular ideal does not satisfy the desired condition. I don't see how it shows there are no other prime ideals satisfying the condition in a regular local ring apart from the family mentioned in the question. $\endgroup$ Feb 7, 2011 at 20:16
  • $\begingroup$ @Koose: I suppose I misunderstood your question. However, the same idea shows that any ideal having this property has to be a complete intersection (that is, it is generated by as many elements as its height). $\endgroup$ Feb 7, 2011 at 21:45
  • $\begingroup$ @Sandor: Thanks for your clarification. I do realize this, but, I was looking for sufficient conditions for an ideal to satisfy the above property. $\endgroup$ Feb 8, 2011 at 0:01

Your Answer

By clicking “Post Your Answer”, you agree to our terms of service and acknowledge you have read our privacy policy.

Not the answer you're looking for? Browse other questions tagged or ask your own question.